Jump to content

Can you move in space?


Recommended Posts

18 hours ago, MPMin said:

This process is happening at an extremely fast rate enabling many pulses of force to occur per second so that even if the force per cycle was very small, the cumulative effect could be a large force.

I think the force would be the same but occur more often.

2 hours ago, MPMin said:

the force per time output

What is that? What are the units? I think you mean something else than N/s (Newton per second)

 

Did you read my last post?

Link to comment
Share on other sites

2 hours ago, MPMin said:

Yes, I was referring to N/s.

I don’t know of any application of N/s in the situations in this thread (or others).

Ns (newton second, equivalent to kgm/s) on the other hand, momentum (or impulse), is important.

What are you attempting to describe using N/s? 

2 hours ago, MPMin said:

Do you think this system could work? 

See my analysis. Ask questions if unclear.

Note: I simplified the and had to catch up on EMPs, there are more room for errrors than usual in that post.

Link to comment
Share on other sites

I was trying to describe the Newton’s of force per second in the same direction or the net Newton’s of force per second. The system might only work on a single cycle as per my first animation or it might even work with the additional cycle which would increase the number of effective impulses hence Newton’s per second. 

I have read your analysis but I don’t fully understand what you are saying. It seems you are saying the system will move in the desired direction. As you haven’t said it will work in a ‘yes or no’ answer I don’t want to assume your conclusion.

The only question I have relating to your analysis is: are you assuming the circular nature of the emp has some kind of structural integrity? The reason I as this is because you seem to say that the force on p has an effect on -p or visa versa (either way, it seems you are saying they are interrelated which suggests to me the circle has an integral structure or that the emp circle emanating away from wire A applies a continual force on wire A even though it’s emanating away from wire A? Another way of thinking of the emp from wire A is imagine a sound wave emanating from wire A, the effect the sound wave has at wire B has no bearing on whats happing on the other side of the sound wave or emp. I hope this makes sense.

im just looking for a conclusive answer with sound reasoning. 

Link to comment
Share on other sites

6 minutes ago, MPMin said:

I was trying to describe the Newton’s of force per second in the same direction or the net Newton’s of force per second.  

This is not a meaningful physics concept in this context.

Link to comment
Share on other sites

4 hours ago, MPMin said:

Im curious about your stand point? 

It can't work because it would violate conservation laws. 

I guess that the explanation would need to take into account the fact that the magnetic pulse from the second wire, B, (can we call them A and B, for simplicity) would induce a current in A and therefore a force on A. If you were to do the full calculations, this (and maybe some second order effects) would exactly counter the force on B.

Also (even if it could work) this is, by definition, is an incredibly inefficient way of generating a force. Just think about it: the magnetic pulse is carrying energy in all directions (360°) away from wire A. A tiny fraction of that field (and energy) intersects with wire B to generate a force. Let's be generous and say that B subtends 0.1° when viewed from A. That means that only 1/3600th of the energy is being used to generate the force. It is hard to imagine a less efficient system.

Link to comment
Share on other sites

On 7/10/2019 at 5:34 AM, MPMin said:

Its hard for me to accept what you are saying when what you are saying is unclear to me.

2 hours ago, MPMin said:

im just looking for a conclusive answer with sound reasoning. 

!

Moderator Note

At the heart of the matter is your lack of physics knowledge, so how can you possibly judge whether an explanation (NOT an answer, science isn't looking for answers) is sound or not? How many times have the members posted something along the lines of "physics doesn't work like that"? Your threads are pages and pages of misunderstandings that could easily be cleared up IF you could put yourself in "learning mode" instead of "I don't understand so it must be wrong" mode. 

It's pretty insulting for you to be questioning the sound reasoning of people who studied this extensively, especially when they're taking their time to help. Doubly especially when basic concepts are met with "Please explain". I think everyone here is more interested in discussing science than they are teaching it, especially in this shambolic style where concepts continue to be misunderstood.

It's great that you're interested, and you're obviously very smart, but how effective do you think this learning style is, where you guess at what you don't understand, and make judgements based on that lack of knowledge and understanding? It seems to generate a lot of questions (which are good) but not a lot of understanding. 

 
2 hours ago, MPMin said:

Please explain why not? 

!

Moderator Note

You ask for explanations but you reject them because you don't understand the underlying, basic physics principles at work. This Catch-22 is hampering your progress, I think. Do you feel as though you're learning anything here? Are you any closer in either supporting or rejecting your idea? You don't have to answer a modnote, but I want you to think about these questions. We're all just interested in discussing the best supported scientific explanations for various phenomena in a meaningful way, and I don't think that's happening here.

 
Link to comment
Share on other sites

2 minutes ago, Strange said:

I guess that the explanation would need to take into account the fact that the magnetic pulse from the second wire, B, (can we call them A and B, for simplicity) would induce a current in A and therefore a force on A. If you were to do the full calculations, this (and maybe some second order effects) would exactly counter the force on B.

Yes lets call the initiating wire (the first wire to fire being the one on the right) A and the other B. Ive thought about this and yes, conventionally, when an emp interacts with a wire it induces a current, but what if that wire is already carrying a current? I believe if that wire is already carrying a current then the emp will induce a force instead. To clarify, If the emp had the potential to induce a current greater than the receiving wire was already carrying, the emp would induce a force to the equivalent magnitude of the current in the receiving wire and the excess emp potential would induce a current in the wire. In other words, if the receiving wire was not carrying a current, 100% of the emp’s power would induce a current in the receiving wire. If the emp had the potential to induce half the current being carried then half the emp’s potential would be converted to Newton’s. Ill try to describe this better latter but essentially the current carried in the receiving wire will act as resistance causing the emp to produce Newtons instead of current in the receiving wire.

Link to comment
Share on other sites

I have just read Ghideon's explanation above. I think it is spot on. 

To summarise: the magnetic field carries momentum. When the pulse leaves A there is zero net momentum because the field is going in all directions and, most importantly for this case, lefthand right.

When the field interacts with B it imparts momentum to the wire. But that momentum comes from the magnetic field (which effectively "disappears" at that point). Now the wire is carrying the momentum to the left while, on the far side, the magnetic field is still carrying the momentum to the right.These are equal and opposite so still zero net momentum  imparted to the craft.

(There will be all sorts of extra effects to consider if you analyse this in detail, like the fact that when B generates its magnetic pulse, it will be moving. And the interaction of this field with wire A. And so on.)

1 minute ago, MPMin said:

Ive thought about this and yes, conventionally, when an emp interacts with a wire it induces a current, but what if that wire is already carrying a current?

The induced current will either add or subtract from that current (which will increase or decrease the force generated). I can't be bothered to work out which way round it is.

We know the idea doesn't work. The challenge is working out why. As you are only proposing a general idea, with no details, then it only has to be refuted in general terms.

Link to comment
Share on other sites

19 minutes ago, Strange said:

We know the idea doesn't work. The challenge is working out why. As you are only proposing a general idea, with no details, then it only has to be refuted in general terms.

What details do you need?

Link to comment
Share on other sites

15 minutes ago, MPMin said:

What details do you need?

We don't need any details. But if you were to provide detailed calculations (currents, length of wires, timing of pulses, distances, etc. etc.) to show exactly how large the force on B is, etc. then those same details could be used to show exactly why the forces/momentum balances out.

However, as you are talking only about the general principle, then we only need to show the general reason that it won't (we have the advantage of knowing that it can't work).

I think that Ghideon has nailed it. 

Link to comment
Share on other sites

35 minutes ago, MPMin said:

Im obviously missing something, Ghideon says the complete rig moves to the left?

Ah, I missed that. I disagree on that point.

However, I can now see an argument why that could be right.The magnetic pulse leaves the craft to the right (*) carrying momentum, and so the craft as a whole gains momentum to the left (the momentum of the right-going part of the pulse having been absorbed by the wire).

(*) And all other directions, but they don't matter here; they all cancel out.

Note that if this line of argument is correct then: 

(a) it has nothing to do with the push on wire B (which is just away of absorbing the momentum of the pulse); and

(b) it is equivalent to something I suggested ages ago: shielding the system in one direction so that the momentum of the pulse can only leave the craft in the other.

 

Let's see what Ghideon and others have to say about it ...

Link to comment
Share on other sites

5 hours ago, Strange said:

Ah, I missed that. I disagree on that point.

It looks like Strange has fully understood the point i try to make, explained it better than me and managed to highlight an issue that i did not see! I will therefore add a few things.

But first: there seems to be an agreement that ”particles carrying momentum” is reasonable model for the pulse as long as no detailed calculations are needed for the interaction at cable B. Therefore that simple model will be used again.

What I missed to explain is that i looked at the best possible outcome only. The interaction at B happens in such a way that all of the incoming momentum P is used to generate an impulse that push at cable B. As Strange pointed out there are other possibilities. Let's draw some and clarify, I’ll draw the complete rig with cables A and B this time. First t=t2.

B141F396-92C9-4E9E-872C-24C047E9CF6C.thumb.jpeg.7bb447f07b3746d81f42bd0f2c5eabbf.jpeg

An electromagnetic pulse, modelled as a set of particles traveling away from cable A is about to reach cable B. Pulse is a circular set of particles but I simplify and draw only particles along X axis carrying momentum P and -P (as I did before)

Next:C42D6CD2-EED8-48B2-A1A0-703FF37FD3C2.thumb.jpeg.436b86434457ee34a8fc9e84e9eda49c.jpeg

First case, the optimal one. B manages to prevent any particles from leaving the system. Exactly how the interaction works is not important. This is the only case i mentioned in the earlier post. Result, due to conservation of momentum is that the rig gains momentum P to the left. As Strange has pointed out several times, an extremely inefficient way to propel the rig compared to use energy to send only pulse -P. P and -P are, also noted by Strange just a tiny part of the energy sent from cable A since A sends the pulse 360 degrees.

next:B208CBC6-5267-49B8-B7D8-B14C28E2D484.thumb.jpeg.8c09e2ffae170f100aa96db73fe2068f.jpeg

Second case, what I think Strange pointed out and correctly expressed disagreement about. This is an interaction with a pulse/field around B in such a way that total momentum of any particles leaving the rig is zero. 

Next:70284A0E-B657-4C5F-8673-4B01C2AE4CB5.thumb.jpeg.0d252cd884e5568caeebacd8ac5466f4.jpeg

Third case supplied as reference; cable B fails to interact at all and particles continue past/through B. Total momentum of pulse leaving system is zero. Rig will not move. 

Again: The system does not ”work”, it may be able to move some if the cable B manages to block some of the pulse to pass and exit the system. Using Strange’s numbers: Cable B does not help the propulsion, it stops it from wasting 3600/3600 of the energy. So instead ”only” 3599/3600 of the energy is wasted. 

The curious reader could re-read the thread and count how many ways the above have already been told, there are several analogies that says the same in this thread. 

Link to comment
Share on other sites

Thank you Ghideon for your very detailed analysis, your drawing will now help me explain my theory even better. I completely agree with everything you said except for one crucial thing, I do not agree that a electromagnetic field pulse (emp) has any momentum ‘unless it interacts with a wire carrying a current’ only then does the emp have momentum at the vicinity of interaction. The reason I believe an emp can have no momentum is because an emp can pass through a non-conductive material having no effect on the non-conductive material and nor does the non-conductive material have any effect on the emp passing through it, hence no apparent momentum in the emp. The momentum of an emp seems to be conditional. The condition to induce momentum from an emp is to obstruct the emp with a wire carrying a current, the wire must be carrying a current at the time of interaction. A wire that’s not carrying a current in the path of an emp, the emp will just induce a current in the wire instead of movement. However, if the wire is carrying a current when the emp interacts with the wire, I believe the current in the wire causes a resistance to the emp which causes the momentum. As the emp is circular, I do not believe this phenomenon will affect the opposite side of the emp (at -p in Ghideon’s drawing) as that part of the emp is not in the vicinity of the current carrying wire (at B and p with reference to Ghideon’s drawing), hence, for all intensive purposes, with reference to Gideon’s drawing, -p always remains 0 (or effectively becomes 0 the further away it is from B and P) as it does not interact with any other magnetic field or current carrying wire on that side, hence -p has no momentum.

Link to comment
Share on other sites

4 hours ago, MPMin said:

I do not agree that a electromagnetic field pulse (emp) has any momentum ‘unless it interacts with a wire carrying a current’ only then does the emp have momentum at the vicinity of interaction.

Please provide a reference. 

 

Link to comment
Share on other sites

5 hours ago, MPMin said:

I do not agree that a electromagnetic field pulse (emp) has any momentum ‘unless it interacts with a wire carrying a current’ only then does the emp have momentum at the vicinity of interaction. 

Obviously wrong.

5 hours ago, MPMin said:

The reason I believe an emp can have no momentum is because an emp can pass through a non-conductive material having no effect on the non-conductive material and nor does the non-conductive material have any effect on the emp passing through it, hence no apparent momentum in the emp. 

You are confusing the momentum of the field with its ability to interact.

Of course the momentum of the field won't be transferred if it doesn't interact with the material. But it will be transferred to a conductor.

We know light has momentum and can push things. However, it will not push a piece of transparent glass (or negligibly so) because it doesn't interact with the glass.

Similarly, tennis balls have momentum and will transfer it to a solid object. But if you throw tennis balls at an object that has lots of holes larger than the balls, then very little momentum will be transferred because the balls won't interact with the object.

 

Link to comment
Share on other sites

You might want to consider how much weight the wiring will add to the craft. As well as the amount of force that would be required to move said craft.

  This I can readily see being a major factor against any practicality of this design

Link to comment
Share on other sites

9 hours ago, Strange said:

Similarly, tennis balls have momentum and will transfer it to a solid object. But if you throw tennis balls at an object that has lots of holes larger than the balls, then very little momentum will be transferred because the balls won't interact with the object.

Yes! and that’s the point isn’t it? Using your analogy, the wire acts as a back board in left direction along the x axis while the other tennis ball rock or emp just goes out the back end affecting nothing in the right direction hence giving a net force to the left.

If it doesn’t have the ability to transfer it’s momentum then it doesn’t transfer it! 

The emp tennis ball rock are thrown simultaneously in opposite directions along the x axis, (we can ignore all the other y directions as Ghidein pointed out ) The two objects being thrown in opposite directions cancel each other out, however in the left direction theres a collision from one of the objects being thrown affecting the movement of the craft to the left while nothing happens to the right - the net effect to the right is 0 because nothing happens in that direction.

To refer back to Ghideons drawing, when p and - p leave the source wire their effect on the source wire are cancelled out by each other, however,  p will collide with B causing a movement to the left while -p does nothing, the net effect of -p is 0 because it doesn’t transfer its momentum to the craft but p does transfer it’s momentum to the craft  moving it to the left.

Edited by MPMin
Link to comment
Share on other sites

2 hours ago, MPMin said:

The emp tennis ball rock are thrown simultaneously in opposite directions along the x axis, (we can ignore all the other y directions as Ghidein pointed out ) The two objects being thrown in opposite directions cancel each other out, however in the left direction theres a collision from one of the objects being thrown affecting the movement of the craft to the left while nothing happens to the right - the net effect to the right is 0 because nothing happens in that direction.

The other directions are important when you think about efficiency.

Your proposal is like throwing rocks in all directions but there is a narrow bar across one end of the craft. That narrow bar stops a tiny number of rocks and so there is a net thrust (because of the excess of rocks in the direction opposite the bar).

So you are wasting energy / fuel by throwing it in all directions, just to generate a tiny thrust because a tiny obstacle blocks a tiny fraction of them.

It would be much more efficient to direct all of the rocks (or all of the electromagnetic pulse) out of the back of the craft.

Link to comment
Share on other sites

Guest
This topic is now closed to further replies.
×
×
  • Create New...

Important Information

We have placed cookies on your device to help make this website better. You can adjust your cookie settings, otherwise we'll assume you're okay to continue.